Number Theory and Euler phi-function

Click For Summary
The discussion centers on proving that a prime number p does not divide a positive integer n if and only if the Euler phi-function satisfies the equation \phi(np) = (p-1)\phi(n). Theorems regarding the properties of the phi-function are referenced, specifically that \phi(p) = p-1 for primes and the multiplicative property of the phi-function for relatively prime integers. The initial assumption that p does not divide n leads to the conclusion that \phi(np) equals (p-1)\phi(n). The challenge arises in proving the reverse implication, particularly in establishing the relative primeness of n and p. The conversation emphasizes the relationship between divisibility and the properties of the phi-function in number theory.
pjc11
Messages
1
Reaction score
0

Homework Statement



Let p be prime. Show that p n, where n is a positive integer, iff \phi(np) = (p-1)\phi(n).

Homework Equations



Theorem 1: If p is prime, then \phi(p) = p-1. Conversely, if p is a positive integer with \phi(p) = p-1, then p is prime.

Theorem 2: Let m and n be relatively prime positive numbers. Then \phi(mn) = \phi(m)\phi(n).

The Attempt at a Solution



Assume that p n.
By Theorem 2, \phi(np) = \phi(n)\phi(p).
By Theorem 1, \phi(np) = \phi(n)\cdot(p-1).
So, if p n, then \phi(np) = (p-1)\cdot\phi(n).

Now, assume that \phi(np) = (p-1)\phi(n).
By Theorem 1, \phi(np) = \phi(p)\phi(n), since p is prime.

...and, that's it. I can't use Theorem 2 to show that n and p are relatively prime, so I'm not entirely sure what to do next. p is prime, so n p, but I don't see anything to base a conclusion of p n.

Any help would be greatly appreciated!
 
Physics news on Phys.org
If a prime p doesn't divide n then aren't p and n relatively prime? A common divisor of p and n must divide p. Think about it.
 
Question: A clock's minute hand has length 4 and its hour hand has length 3. What is the distance between the tips at the moment when it is increasing most rapidly?(Putnam Exam Question) Answer: Making assumption that both the hands moves at constant angular velocities, the answer is ## \sqrt{7} .## But don't you think this assumption is somewhat doubtful and wrong?

Similar threads

  • · Replies 9 ·
Replies
9
Views
2K
  • · Replies 2 ·
Replies
2
Views
3K
  • · Replies 13 ·
Replies
13
Views
4K
  • · Replies 1 ·
Replies
1
Views
2K
  • · Replies 3 ·
Replies
3
Views
2K
Replies
7
Views
3K
Replies
4
Views
2K
  • · Replies 10 ·
Replies
10
Views
1K
Replies
8
Views
2K
  • · Replies 1 ·
Replies
1
Views
2K